Rolling coin

A U.S quarter is rolling on the floor without slipping in such a way that it describes a circular path of radius R = 4 cm R=4~\mbox{cm} . The plane of the coin is tilted at an angle of θ = 4 5 \theta=45^{\circ} with respect to the horizontal plane (see the figure below). Find the coin's period T T in seconds , that is, the time it takes for the coin to go around the circle of radius R R . The radius of a U.S quarter is r = 1.2 cm r=1.2 ~\mbox{cm} .

Details and assumptions

Assume g = 9.8 m/s 2 g=9.8~\mbox{m/s}^{2} .


The answer is 0.446.

This section requires Javascript.
You are seeing this because something didn't load right. We suggest you, (a) try refreshing the page, (b) enabling javascript if it is disabled on your browser and, finally, (c) loading the non-javascript version of this page . We're sorry about the hassle.

2 solutions

Leonardo Jackson
Aug 1, 2013

It would be quite difficult to explain the answer to this question without images so i would be giving links to images I used for solving the question:

So a simplified figure for starting off with the solution:

Let's first go to the CM frame. As it can be seen in the figure above, I will illustrate the principal axis by x 1 x_1 (into the page), x 2 x_2 and x 3 x_3 .

Let Ω \Omega be the angular velocity of the attaching point of the coin with the floor around the center. This means T T = 2 π Ω 2\pi\Omega .

Using the non-slipping condition, one will find out that the total angular momentum is:

ω = Ω z ^ R r Ω x ^ 3 \vec{\omega}=\Omega \hat{z}- \frac{R}{r}\Omega \hat{x}_3

Now writing z ^ \hat{z} in the principal coordinates as sin θ x ^ 2 + cos θ x ^ 3 \sin{\theta}\hat{x}_2+\cos{\theta}\hat{x}_3 we will have:

ω = Ω sin θ x ^ 2 Ω ( R r cos θ ) x ^ 3 \vec{\omega} = \Omega \sin{\theta} \hat{x}_2 - \Omega \left(\frac{R}{r}-\cos{\theta} \right)\hat{x}_3

The principal moments are:

I 1 = I 2 = m r 2 4 and I 3 = m r 2 2 I_1=I_2=\frac{mr^2}{4} \ \text{ and } \; I_3=\frac{mr^2}{2}

so the angular momentum will be:

L = m r 2 2 ( 1 2 Ω sin θ x ^ 2 Ω ( R r cos θ ) x ^ 3 ) \vec{L}= \frac{mr^2}{2}\left(\frac12\Omega \sin{\theta} \hat{x}_2 - \Omega \left(\frac{R}{r}-\cos{\theta} \right)\hat{x}_3\right)

Only the horizontal component of L \vec{L} is changing:

d L d t = Ω L = 1 4 m r Ω 2 sin θ ( 2 R r cos θ ) . |\frac{d \vec{L}}{dt}|=\Omega L_{\perp}=\frac{1}{4}mr\Omega^2\sin\theta\left( 2R-r\cos\theta \right).

Now we have to calculate the torque relative to CM, as well. The torque comes from the forces at the contact point, the horizontal force is m ( R r cos θ ) Ω 2 m(R-r\cos\theta)\Omega^2 and the vertical force will be simply m g mg . :

τ = m g ( r cos θ ) m ( R r cos θ ) Ω 2 ( r sin θ ) |\vec \tau|= mg(r \cos θ) - m(R - r \cos θ)Ω^2 (r \sin θ)

Using τ = d L d t |\vec\tau|=|\frac{d\vec L}{dt}| , we can find:

Ω = 2 g 6 R tan θ 5 r sin θ \Omega=2\sqrt{\frac{g}{6 R \tan\theta - 5 r \sin{\theta}}}

T = π 6 R tan θ 5 r sin θ g 0.446 s \Rightarrow T= \pi \sqrt{\frac{6 R \tan\theta - 5 r \sin{\theta}}{g}} \approx 0.446 \text{s}

http://physics.stackexchange.com/questions/72607/a-rolling-quarter

Louie Tan Yi Jie - 7 years, 10 months ago

Log in to reply

see before writing !!!

Leonardo Jackson - 7 years, 10 months ago

I didn't understand properly

Kiran Rajput - 7 years, 10 months ago

Log in to reply

which part ?

Leonardo Jackson - 7 years, 10 months ago

Hey its a cheated solution from physicsstackechange...

You are right Louie T

Kiran Rajput - 7 years, 10 months ago

interesting so r u 21 and in UMD study the quantum physic and then also 14 inside of India??? here is ali: http://groups.jqi.umd.edu/gorshkov/people/ali-hamed-moosavianexpected

more information of ali: http://physics.stackexchange.com/users/24791/ali

Jindus Mayad - 7 years, 10 months ago

Log in to reply

can't it be ?

Leonardo Jackson - 7 years, 10 months ago

Obviously, the solution is copied. Who posted the problem on stack exchange anyways?

PS: Nice to know someone from the same city.

Pranav Arora - 7 years, 10 months ago

Perfect and clear !

Ritvik Choudhary - 7 years, 10 months ago

Can you please explain how you wrote down the expression for omega

Navin Murarka - 4 years ago

Hey everybody listen i'm not that kind of guy that's me only on stack exchange with my other name of Ali so that's only me here so please think before writing anything for anybody !!!

Leonardo Jackson - 7 years, 10 months ago

Log in to reply

should edit stack exchange post for some way to know is true

Jindus Mayad - 7 years, 10 months ago

Damned big time..

Nishanth Hegde - 7 years, 10 months ago
Tan Kiat
Mar 15, 2015

Let us consider the triangle relating the base of the quarter, B , center of gravity of the quarter, G, and the center of the circle, C.

Note that angle GBC = 4 5 o = 45^o due to the tilt of the quarter, and lengths B C BC and B G BG are R = 4 R = 4 cm and r = 1.2 r = 1.2 cm respectively. Hence, using cosine rule, length C G = 3.2637 c m CG = 3.2637 cm . Thereafter, using sine rule, angle GCB = 15.06 9 o = 15.069^o

Now, we know that the centripetal acceleration required must be along line C G CG directed at an angle of 15.06 9 o 15.069^o from the horizontal.

Let us now consider the free body diagram of the coin. At the center of gravity, we have the acceleration due to normal contact force , a N a_N , and the acceleration due to the weight of the coin g g . a N a_N is directed 4 5 o 45^o away from the horizontal. Now, we need to resolve the forces to produce one that is along line C G CG

We now set line CG as the new x-axis and its corresponding normal as y-axis, which this new set of orthogonal base tilted 15.06 9 o 15.069^o counter-clockwise. Now, the angle that g g makes with the y-axis is 15.06 9 o 15.069^o and the angle that a N a_N makes with the y-axis is 4 5 o 15.06 9 o = 29.93 1 o 45^o - 15.069^o = 29.931^o

With that, resolving forces along y-axis,

a N c o s ( 29.93 1 o ) = g c o s ( 15.06 9 o ) a_N cos(29.931^o) = g cos(15.069^o) ---- Equation (1)

Resolving along x-axis,

a c = a N s i n ( 29.93 1 o ) + g s i n ( 15.06 9 o ) a_c = a_N sin(29.931^o) + g sin(15.069^o) --- Equation (2)

Hence, we can solve for a c a_c

Using a c = r ω 2 a_c = r\omega^2 , and ω = 2 π T \omega = \dfrac{2\pi}{T} ,

T = 0.444 s T = \boxed{0.444s}

Note : The answer might be slightly off due to round-off errors :X

0 pending reports

×

Problem Loading...

Note Loading...

Set Loading...